Exos sympas MP(*)

Un problème, une question, un nouveau théorème ?

Messages : 172

Inscription : 11 mars 2021 18:24

Profil de l'utilisateur : Élève de lycée

Re: Exos sympas MP(*)

Message par Contrexemple » 24 févr. 2022 00:03

Calculer, en le justifiant $$U=\sum \limits _{k \geq 1,\gcd(k,30)=1} \dfrac {1}{k^2}$$.


Théorème de convergence monotone :
$ $
Soit $f_n$ suite de fonctions croissantes sur I=[0,1] qui converge simplement, dans $\mathbb R \cup \{-\infty, +\infty \} $ vers $f_\infty $ avec :
toutes ces fonctions Riemann intégrables et $\forall x \in [0,1], f_n(x) \leq f_{n+1}(x)$.

A-t-on $\lim \int_0^1 f_n(x) dx=\int_0^1 \lim f_n(x) dx$?

Ps : on répondra sans utiliser le théorème de convergence dominée.

Messages : 172

Inscription : 11 mars 2021 18:24

Profil de l'utilisateur : Élève de lycée

Re: Exos sympas MP(*)

Message par Contrexemple » 27 févr. 2022 15:12

Interversion de limite

Soit $a_{n, m} $ suite double de réels borné, tel que pour toutes fonctions $f, g$ des entiers dans les entiers, strictement croissantes, $a_{f(n), g(n)} $ converge.

A-t-on $\lim\limits_{n} \lim\limits_{m} a_{n, m} =\lim\limits_{m}\lim\limits_{n} a_{n, m} $, on supposera chaque limite bien définie ?

Messages : 172

Inscription : 11 mars 2021 18:24

Profil de l'utilisateur : Élève de lycée

Re: Exos sympas MP(*)

Message par Contrexemple » 03 mars 2022 19:38

Soit $f$ une application linéaire de E $\mathbb R$ev de dim n, dans E, tel que :

$$\forall x\in E, \exists k\in\mathbb N, f^k(x) =0$$

A-t-on $f$ nilpotente?

Messages : 172

Inscription : 11 mars 2021 18:24

Profil de l'utilisateur : Élève de lycée

Re: Exos sympas MP(*)

Message par Contrexemple » 04 mars 2022 11:20

Contrexemple a écrit :
24 févr. 2022 00:03
Calculer, en le justifiant $$U=\sum \limits _{k \geq 1,\gcd(k,30)=1} \dfrac {1}{k^2}$$.
Résolue par AOPS : https://artofproblemsolving.com/communi ... late_serie

Messages : 1

Inscription : 30 juil. 2021 14:37

Profil de l'utilisateur : Élève de lycée

Re: Exos sympas MP(*)

Message par Pré 7/2 » 04 mars 2022 11:27

Contrexemple a écrit :
03 mars 2022 19:38
Soit <span class="MathJax_Preview" style="color: inherit; display: none;"></span><span id="MathJax-Element-4-Frame" class="mjx-chtml MathJax_CHTML" tabindex="0" data-mathml="<math xmlns=&quot;http://www.w3.org/1998/Math/MathML&quot ... /mi></math>" role="presentation" style="font-size: 121%; position: relative;"><span id="MJXc-Node-12" class="mjx-math" aria-hidden="true"><span id="MJXc-Node-13" class="mjx-mrow"><span id="MJXc-Node-14" class="mjx-mi"><span class="mjx-char MJXc-TeX-math-I" style="padding-top: 0.455em; padding-bottom: 0.455em; padding-right: 0.06em;">f</span></span></span></span><span class="MJX_Assistive_MathML" role="presentation"><math xmlns="http://www.w3.org/1998/Math/MathML"><mi ... an><script type="math/tex" id="MathJax-Element-4">f</script> une application linéaire de E <span class="MathJax_Preview" style="color: inherit; display: none;"></span><span id="MathJax-Element-5-Frame" class="mjx-chtml MathJax_CHTML" tabindex="0" data-mathml="<math xmlns=&quot;http://www.w3.org/1998/Math/MathML&quot;><mrow class=&quot;MJX-TeXAtom-ORD&quot;><mi mathvariant=&quot;double-struck&quot;>R</mi></mrow></math>" role="presentation" style="font-size: 121%; position: relative;"><span id="MJXc-Node-15" class="mjx-math" aria-hidden="true"><span id="MJXc-Node-16" class="mjx-mrow"><span id="MJXc-Node-17" class="mjx-texatom"><span id="MJXc-Node-18" class="mjx-mrow"><span id="MJXc-Node-19" class="mjx-mi"><span class="mjx-char MJXc-TeX-ams-R" style="padding-top: 0.455em; padding-bottom: 0.326em;">R</span></span></span></span></span></span><span class="MJX_Assistive_MathML" role="presentation"><math xmlns="http://www.w3.org/1998/Math/MathML"><mrow class="MJX-TeXAtom-ORD"><mi mathvariant="double-struck">R</mi></mrow></math></span></span><script type="math/tex" id="MathJax-Element-5">\mathbb R</script>ev de dim n, dans E, tel que :

<mjx-container class="MathJax CtxtMenu_Attached_0" jax="CHTML" display="true" tabindex="0" ctxtmenu_counter="3" style="font-size: 113.1%; position: relative;"><mjx-math display="true" class="MJX-TEX" aria-hidden="true" style="margin-left: 0px; margin-right: 0px;"><mjx-mi class="mjx-n"><mjx-c class="mjx-c2200"></mjx-c></mjx-mi><mjx-mi class="mjx-i"><mjx-c class="mjx-c1D465 TEX-I"></mjx-c></mjx-mi><mjx-mo class="mjx-n" space="4"><mjx-c class="mjx-c2208"></mjx-c></mjx-mo><mjx-mi class="mjx-i" space="4"><mjx-c class="mjx-c1D438 TEX-I"></mjx-c></mjx-mi><mjx-mo class="mjx-n"><mjx-c class="mjx-c2C"></mjx-c></mjx-mo><mjx-mi class="mjx-n" space="2"><mjx-c class="mjx-c2203"></mjx-c></mjx-mi><mjx-mi class="mjx-i"><mjx-c class="mjx-c1D458 TEX-I"></mjx-c></mjx-mi><mjx-mo class="mjx-n" space="4"><mjx-c class="mjx-c2208"></mjx-c></mjx-mo><mjx-texatom space="4" texclass="ORD"><mjx-mi class="mjx-ds mjx-b"><mjx-c class="mjx-c2115 TEX-A"></mjx-c></mjx-mi></mjx-texatom><mjx-mo class="mjx-n"><mjx-c class="mjx-c2C"></mjx-c></mjx-mo><mjx-msup space="2"><mjx-mi class="mjx-i"><mjx-c class="mjx-c1D453 TEX-I"></mjx-c></mjx-mi><mjx-script style="vertical-align: 0.413em; margin-left: 0.053em;"><mjx-mi class="mjx-i" size="s"><mjx-c class="mjx-c1D458 TEX-I"></mjx-c></mjx-mi></mjx-script></mjx-msup><mjx-mo class="mjx-n"><mjx-c class="mjx-c28"></mjx-c></mjx-mo><mjx-mi class="mjx-i"><mjx-c class="mjx-c1D465 TEX-I"></mjx-c></mjx-mi><mjx-mo class="mjx-n"><mjx-c class="mjx-c29"></mjx-c></mjx-mo><mjx-mo class="mjx-n" space="4"><mjx-c class="mjx-c3D"></mjx-c></mjx-mo><mjx-mn class="mjx-n" space="4"><mjx-c class="mjx-c30"></mjx-c></mjx-mn></mjx-math><mjx-assistive-mml unselectable="on" display="block"><math xmlns="http://www.w3.org/1998/Math/MathML" display="block"><mi mathvariant="normal">∀</mi><mi>x</mi><mo>∈</mo><mi>E</mi><mo>,</mo><mi mathvariant="normal">∃</mi><mi>k</mi><mo>∈</mo><mrow data-mjx-texclass="ORD"><mi mathvariant="double-struck">N</mi></mrow><mo>,</mo><msup><mi>f</mi><mi>k</mi></msup><mo stretchy="false">(</mo><mi>x</mi><mo stretchy="false">)</mo><mo>=</mo><mn>0</mn></math></mjx-assistive-mml></mjx-container>

A-t-on <span class="MathJax_Preview" style="color: inherit; display: none;"></span><span id="MathJax-Element-6-Frame" class="mjx-chtml MathJax_CHTML" tabindex="0" data-mathml="<math xmlns=&quot;http://www.w3.org/1998/Math/MathML&quot ... /mi></math>" role="presentation" style="font-size: 121%; position: relative;"><span id="MJXc-Node-20" class="mjx-math" aria-hidden="true"><span id="MJXc-Node-21" class="mjx-mrow"><span id="MJXc-Node-22" class="mjx-mi"><span class="mjx-char MJXc-TeX-math-I" style="padding-top: 0.455em; padding-bottom: 0.455em; padding-right: 0.06em;">f</span></span></span></span><span class="MJX_Assistive_MathML" role="presentation"><math xmlns="http://www.w3.org/1998/Math/MathML"><mi ... an><script type="math/tex" id="MathJax-Element-6">f</script> nilpotente?
SPOILER:
On remarque qu'un tel entier k est nécessairement plus petit que n quitte à considérer la famille (x,f(x),...,f^k-1(x)) libre en composant successivement par f^k-1,f^k-2,..., une relation de liaison de cette famille. Elle est donc de cardinal plus petit que n.

On écrit alors que E est la réunion des Ker(f^k) pour k allant de 1 à n, et une union finie de sev sur un corps K est un ev à condition que l'un de ces sev contienne tous les autres (preuve par récurrence sur un corps K quelconque qui contient plus d'éléments que le nombre de sev supposément non inclus les uns dans les autres (ici R est infini), ou, plus simplement ici car on est sur un R-ev de dimension finie, une preuve topologique: des sev stricts sont des fermés d'intérieur vide et une union finie de fermés d'intérieur vide est d'intérieur vide).

Un des Ker(f^k) contient alors tous les autres et est égal à E, c'est-à-dire dire que f est nilpotente.

Messages : 172

Inscription : 11 mars 2021 18:24

Profil de l'utilisateur : Élève de lycée

Re: Exos sympas MP(*)

Message par Contrexemple » 04 mars 2022 11:44

@Pré 7/2 :
SPOILER:
$ $Ta première phrase suffit pour conclure : en effet tu sais que pour tout $x$ dans E, il existe $k\leq n$ tel que $f^k(x)=0$ comme $f^{n-k}$ est linéaire, on a $f^{n-k}(f^k(x))=0$ c'est à dire $f^n(x)=0$

Messages : 172

Inscription : 11 mars 2021 18:24

Profil de l'utilisateur : Élève de lycée

Re: Exos sympas MP(*)

Message par Contrexemple » 04 mars 2022 13:23

Contrexemple a écrit :
23 nov. 2021 15:01
Soit $p>5$, $H$sous groupe de $(\mathbb Z/p\mathbb Z) ^*$, avec $a\in H$ et $a>2$.

A-t-on $(a-1)| p\sum\limits_{h \in H} (-h/p \mod a) $?
Résolue par la communauté mathoverflow :

https://mathoverflow.net/questions/4173 ... visibility

Si vous avez le niveau agreg et que vous voulez quelques choses de difficiles :

https://mathoverflow.net/questions/2739 ... it-abelian

https://mathoverflow.net/questions/3534 ... l-of-sqrt2


PS : je ne sais qu'un énoncé est difficile que lorsqu'il résiste au moins un an à la sagacité des mathématiciens de mathoverflow.

Par exemple celui-ci à résister un an à aops et moins d'une heure à mathoverflow.

Messages : 172

Inscription : 11 mars 2021 18:24

Profil de l'utilisateur : Élève de lycée

Re: Exos sympas MP(*)

Message par Contrexemple » 04 mars 2022 22:21

Contrexemple a écrit :
15 févr. 2022 12:02
Soit $p$ entier premier, $f$ une fonction de $A=\mathbb F_p^n$ dans $\mathbb F_p$, avec $n\geq p+1$.

A-t-on pour toutes $x\in A, \sum\limits_{\sigma \in S_n} s(\sigma) \times f(x_\sigma) =0$?

$s$ la signature
https://mathoverflow.net/questions/4174 ... still-zero

Messages : 172

Inscription : 11 mars 2021 18:24

Profil de l'utilisateur : Élève de lycée

Re: Exos sympas MP(*)

Message par Contrexemple » 07 mars 2022 14:30

Salut,

Un nouveau que je trouve jolie et qui peut s'avérer trés utile.

Soit $P,Q \in \mathbb Z[x]$ avec $\gcd(P(x),Q(x))=1$.

A-t-on alors il existe $N \in \mathbb N$ tel que : $\forall n\in \mathbb N, \dfrac{N}{\gcd(P(n),Q(n))} \in \mathbb N$ ?

Messages : 41

Inscription : 22 août 2018 15:42

Profil de l'utilisateur : Élève de lycée

Re: Exos sympas MP(*)

Message par GaBuZoMeu » 07 mars 2022 15:04

Bonjour,

Oui, trivialement : $ N=0 $.

Mais c'est vrai encore quand on corrige l'énoncé en demandant $ N\neq 0 $.
Si 1 est pgcd de $ P $ et $ Q $ dans $ \mathbb Z[X] $, alors $ P $ et $ Q $ sont premiers entre eux dans $ \mathbb Q[X] $ et il existe donc $ U,V $ dans $ \mathbb Z[X] $ et $ N\in \mathbb N^* $ tels que $ UP+VQ=N $.

Répondre